Đến nội dung

phuc_90 nội dung

Có 79 mục bởi phuc_90 (Tìm giới hạn từ 29-03-2020)



Sắp theo                Sắp xếp  

#730812 Chứng minh định lý lớn Fermat ngắn gọn bằng toán sơ cấp

Đã gửi bởi phuc_90 on 30-09-2021 - 14:48 trong Nghiên cứu Toán học

Đọc phần chứng minh này tôi thấy bạn đang đi tìm giá trị để các cạnh của một tam giác là số nguyên. Chứ chưa thấy giải quyết được gì.

 

Từ  $z<x+y$ và $z>y>x$ mà suy ra đó là điều kiện để $x^n+y^n=z^n$ có nghiệm với $n\geq 2$ là hết sức phi lý rồi.




#730127 Đề thi thử đội tuyển THPT Ngô Gia Tự 2021 -2022

Đã gửi bởi phuc_90 on 04-09-2021 - 18:12 trong Thi HSG cấp Tỉnh, Thành phố. Olympic 30-4. Đề thi và kiểm tra đội tuyển các cấp.


Ngày 2

 

8. Cho $n$ là một số nguyên dương. Hãy tìm số nguyên $k$ nhỏ nhất với tính chất sau:

Với mọi số thực $a_1,...,a_d$ sao cho $a_1+a_2+...+a_d=n$ và $0\le a_i\le 1$ $(i=\overline{1,d})$, ta luôn có thể chia $d$ số trên thành $k$ nhóm, và tổng các số trong mỗi nhóm không lớn hơn $1$.

 

 

Trước tiên ta đặt $S_d=\{a_1,a_2,...,a_d\}$ và $sumS_d=a_1+a_2+...+a_d$

 

Giả sử với $d$ số đã cho như trong đề bài, ta luôn có thể chia $d$ số đó thành $k$ nhóm mà tổng các số trong mỗi nhóm không lớn hơn $1$

 

Gọi các nhóm đó là $S_1, S_2,...,S_k$ với $sumS_i \leq 1$ $(i=\overline{1,k})$

 

Từ đó ta có thể suy ra $S_d=\bigcup_{i=1}^{k}S_i$ suy ra $n=sumS_d=\sum_{i=1}^{k}sumS_i \leq k$

 

Từ những điều trên ta thấy $k=n$ là số nguyên nhỏ nhất thỏa mãn đề bài

 

Bây giờ với $k=n$ ta sẽ chứng minh tồn tại cách chia $d$ số thỏa $a_1+a_2+...+a_d=n$ và $0\le a_i\le 1$ $(i=\overline{1,d})$ thành $n$ nhóm sao cho tổng các số trong mỗi nhóm không lớn hơn $1$ (*)

 

Với $n=1$ thì hiển nhiên có 1 nhóm thỏa đề bài đó là $\{a_1,a_2,...,a_d\}$

 

Giả sử điều này đúng với $n=k$, khi đó với $a_1+a_2+...+a_d=k+1>k$ (**)

 

Khi đó tồn tại tổng $a_{i_1}+a_{i_2}+...+a_{i_q}$ là tổng nhỏ hơn $1$ lớn nhất với số phần tử của tổng bé hơn $d$

 

Điều này dẫn đến $2\geq a_{i_1}+a_{i_2}+..+a_{i_q}+a_{i_{q+1}}>1$, đặt $b=a_{i_1}+a_{i_2}+..+a_{i_q}+a_{i_{q+1}}-1$ suy ra $0<b\leq 1$

 

 

Khi đó (**) trở thành $a_1+a_2+...+b+...+a_d=k$, theo giả thiết qui nạp thì ta có thể chia thành $k$ nhóm mà tổng các số trong mỗi nhóm không lớn hơn $1$

 

Giả sử $b\in S_i$, ta bỏ phần tử $b$ ra khỏi tập $S_i$ rồi lập thành 2 tập mới đó là $S'_i=S_i\setminus b$ và $K=\{a_{i_1},a_{i_2},...,a_{i_q}\}$

 

Rõ ràng các tập này có $sumS'_i\leq 1$ và $sumK\leq 1$ và $S_d=S_1\bigcup S_2 \bigcup ... \bigcup S'_i \bigcup K \bigcup ... \bigcup S_k$

 

hay $S$ được chia thành $k+1$ nhóm con mà tổng các số của mỗi nhóm không lớn hơn $1$




#730126 Đề thi thử đội tuyển THPT Ngô Gia Tự 2021 -2022

Đã gửi bởi phuc_90 on 04-09-2021 - 17:26 trong Thi HSG cấp Tỉnh, Thành phố. Olympic 30-4. Đề thi và kiểm tra đội tuyển các cấp.


Ngày 2

 

5. Cho ba số thực $x$, $y$, $z$ là 3 số thực không âm thỏa mãn $x^{2}+y^{2}+z^{2}=1$. Chứng minh:

$T= \frac{x}{1-yz}+\frac{y}{1-zx}+\frac{z}{1-xy}$ chỉ có thể nhận tối đa 2 giá trị nguyên. Tìm các bộ $(x,y,z)$ để $T$ đạt giá trị tại 2 giá trị nguyên này

 

Ta có $2-2x+2xyz=1+x^2+y^2+z^2-2x+2xyz=(x-1)^2+y^2+z^2+2xyz \geq 0$ $\Rightarrow x(1-yz)\leq 1$

 

Khi đó $\frac{x}{1-yz}=\frac{x^2}{x(1-yz)}\geq x^2$

 

Lập luận tương tự ta cũng có $\frac{y}{1-zx}=\frac{y^2}{y(1-zx)}\geq y^2$  và  $\frac{z}{1-xy}=\frac{z^2}{z(1-xy)}\geq z^2$

 

Suy ra $P\geq x^2+y^2+z^2=1$

 

Ta lại có $2-2yz\geq 2-(y^2+z^2)=1+x^2=\frac{1}{3}+\frac{1}{3}+\frac{1}{3}+x^2\geq \frac{4}{\sqrt[4]{27}}\sqrt{x}$ $\Rightarrow \frac{x}{1-yz}\leq \frac{\sqrt[4]{27}}{2}\sqrt{x}$

 

Lập luận tương tự ta cũng có $\frac{y}{1-zx}\leq \frac{\sqrt[4]{27}}{2}\sqrt{y}$ và $\frac{z}{1-xy}\leq \frac{\sqrt[4]{27}}{2}\sqrt{z}$

 

Suy ra $P\leq \frac{\sqrt[4]{27}}{2}(\sqrt{x}+\sqrt{y}+\sqrt{z})$

 

Theo bất đẳng thức C-S thì $\sqrt{x}+\sqrt{y}+\sqrt{z}\leq \sqrt{3(x+y+z)}\leq \sqrt{3\sqrt{3(x^2+y^2+z^2)}}=\sqrt[4]{27}$

 

Suy ra $P\leq \frac{3\sqrt{3}}{2}<3$

 

Vậy $P$ chỉ nhận tối đa 2 giá trị nguyên là $1$ và $2$

 

Khi $P=1$ thì ta dễ dàng tìm được bộ $(x,y,z)$ thỏa mãn giá trị của P đó là $(1,0,0), (0,1,0),(0,0,1)$

 

Khi $P=2$ thì ta tìm được bộ $(x,y,z)$ đó là $(\sqrt{1-2t^2}, t, t)$ và các hoán vị của nó

 

trong đó $t$ chính là nghiệm của phương trình $8t^8-20t^6-8t^5+20t^4+16t^3-9t^2-8t+3=0$

 

Note:   Phương trình trên có 2 nghiệm thực $t_1\approx 0,351078$ và $t_2\approx 0,692448$




#730532 Chứng minh a.1 = a

Đã gửi bởi phuc_90 on 19-09-2021 - 14:48 trong Tổ hợp và rời rạc

0.1=0; 1.1 tại sao lại bằng 1

Tôi hỏi bạn $1.1 = ?$ , sao bạn lại đi hỏi ngược lại tôi.




#730528 Chứng minh a.1 = a

Đã gửi bởi phuc_90 on 19-09-2021 - 12:43 trong Tổ hợp và rời rạc

Chứng minh a.1= a

a, Phương pháp biến đổi đại số

b, Phương pháp phản chứng.

 

Cho hỏi $0.1 = ? \,\,,\,\,1.1 = ?$ rồi tính tiếp :lol:




#730489 Tìm a;b để tồn tại k nguyên dương thỏa $a_{0}>a_{1...

Đã gửi bởi phuc_90 on 17-09-2021 - 15:58 trong Dãy số - Giới hạn

Cho dãy số $a_{n}$ xác định như sau: $a_{0}=a;a_{1}=b$ và $a_{n+2}=5a_{n+1}-a_{n}$ biết a là các số nguyên dương. Tìm điều kiện của a;b để tồn tại một chỉ số k ( k nguyên dương) sao cho $a_{0}>a_{1}>a_{2}>...>a_{k-1}>a_{k}=1$

 

$a=9$ ,  $b=2$  thì $k=2$




#730502 Tìm a;b để tồn tại k nguyên dương thỏa $a_{0}>a_{1...

Đã gửi bởi phuc_90 on 18-09-2021 - 12:32 trong Dãy số - Giới hạn

Cho dãy số $a_{n}$ xác định như sau: $a_{0}=a;a_{1}=b$ và $a_{n+2}=5a_{n+1}-a_{n}$ biết a là các số nguyên dương. Tìm tất cả điều kiện của a;b để tồn tại một chỉ số k ( k nguyên dương) sao cho $a_{0}>a_{1}>a_{2}>...>a_{k-1}>a_{k}=1$

P/S: Bài này em có dùng thử dùng phương trình đặc trưng nhưng số xấu quá nên nhờ mọi người xem giúp.

 

Post bài toán gốc lên đi em trai (đọc bài toán này cũng không hiểu em muốn hỏi cái gì)
 




#730270 Tính lim $x_{n+1}=\sqrt{3}+\frac{x_n...

Đã gửi bởi phuc_90 on 09-09-2021 - 23:05 trong Dãy số - Giới hạn

Bạn chứng minh được nó hội tụ không

 

Đặt $f(x)=\sqrt{3}+\frac{x}{\sqrt{x^2-1}}$  ta có $f(x)$ liên tục trên $\mathbb{R^+}\setminus \{\pm 1\}$

 

và $f'(x)=-\frac{1}{(x^2-1)\sqrt{x^2-1}}<0$ suy ra $f(x)$ là hàm nghịch biến trên $\mathbb{R^+}\setminus \{\pm 1\}$

 

Với $x_1=2006$ suy ra $x_2\approx 2.73$ và $x_3\approx 2.81$

 

Khi đó $x_1>x_3 \Rightarrow x_2=f(x_1)<f(x_3)=x_4 \Rightarrow x_3=f(x_2)>f(x_4)=x_5$

 

Bằng phương pháp qui nạp ta chứng minh được $(x_{2n+1})$ là dãy giảm và bị chặn dưới bởi $1+\sqrt{3}$

 

$(x_{2n})$ là dãy tăng và bị chặn trên bởi $\sqrt{3}+\sqrt{\frac{4+2\sqrt{3}}{3+2\sqrt{3}}}$

 

Khi đó giới hạn của dãy $(x_{2n+1})$, $(x_{2n})$ tồn tại, đặt $\lim_{n\rightarrow \infty }x_{2n+1}=a$ và $\lim_{n\rightarrow \infty }x_{2n}=b$

 

Từ đó ta tìm nghiệm của hệ $$\left\{\begin{matrix}a=\sqrt{3}+\frac{a}{\sqrt{a^2-1}}\\ b=\sqrt{3}+\frac{b}{\sqrt{b^2-1}}\end{matrix}\right.$$  với $a\geq 1+\sqrt{3}$ và $1+\sqrt{3}<b\leq \sqrt{3}+\sqrt{\frac{4+2\sqrt{3}}{3+2\sqrt{3}}}$

 

Hệ có nghiệm $a=b=\frac{1}{2}\left ( \sqrt{3}+\sqrt{15} \right )$

 

Suy ra $\lim_{n\rightarrow \infty }x_{n}=\frac{1}{2}\left ( \sqrt{3}+\sqrt{15} \right )$




#730171 Đề thi thử đội tuyển Olympic 30-4 THPT Ngô Gia Tự - Khối 11

Đã gửi bởi phuc_90 on 05-09-2021 - 23:09 trong Thi HSG cấp Tỉnh, Thành phố. Olympic 30-4. Đề thi và kiểm tra đội tuyển các cấp.

1. Cho $a, b, c$ là các số thực không âm. Chứng minh rằng $(a+bc)^{2}+(b+ca)^{2}+(c+ab)^{2}\geq \sqrt{2} (a+b)(b+c)(c+a)$

 

 

Một trong ba số $(1-a)(1-b)$, $(1-b)(1-c)$, $(1-c)(1-a)$ phải có ít nhất một số dương, nếu không có số nào dương thì tích 3 số này là một số âm (vô lý). Không mất tính tổng quát, giả sử $(1-b)(1-c)\geq 0$

 

Theo bất đẳng thức Cauchy-Schwarz ta có  $$(b+ca)^2+(c+ab)^2\geq \frac{(b+ca+c+ab)^2}{2}=\frac{(1+a)^2(b+c)^2}{2}$$

 

Theo bất đẳng thức AM-GM thì $$\frac{(1+a)^2(b+c)^2}{2}+(a+bc)^2\geq \sqrt{2}(1+a)(b+c)(a+bc)$$

 

Từ những điều trên ta suy ra $VT\geq \sqrt{2}(1+a)(b+c)(a+bc)$

 

Ta lại có $(1+a)(b+c)(a+bc)-(a+b)(b+c)(c+a)=a(1-b)(1-c)(b+c)\geq 0$

 

suy ra $(1+a)(b+c)(a+bc)\geq (a+b)(b+c)(c+a)$

 

Vậy $VT\geq \sqrt{2}(a+b)(b+c)(c+a) $




#730196 Đề thi thử đội tuyển Olympic 30-4 THPT Ngô Gia Tự - Khối 11

Đã gửi bởi phuc_90 on 06-09-2021 - 21:45 trong Thi HSG cấp Tỉnh, Thành phố. Olympic 30-4. Đề thi và kiểm tra đội tuyển các cấp.


5. Sau khi cân $11$ con gà, bác nông dân có nhận xét sau:

- Luôn có thể chia $10$ con gà bất kì thành hai nhóm, mỗi nhóm $5$ con, sao cho tổng cân nặng ở mỗi nhóm bằng nhau

- Tổng cân nặng của cả đàn gà là $759$.

Tính cân nặng của mỗi con gà, biết rằng các cân nặng này đều là số tự nhiên.

Gọi $a_1, a_2, ..., a_{11}$ là cân nặng của mỗi con gà. Khi đó với 2 bộ $(a_1,a_2,...,a_{10}), (a_1,a_2,...,a_8,a_9,a_{11})$ mà mỗi bộ được chia thành 2 nhóm bất kì nên ta có thể chia thành

 

$$\left\{\begin{matrix}a_1+a_2+a_3+a_4+a_5=a_6+a_7+a_8+a_9+a_{10}\\a_1+a_2+a_3+a_4+a_5=a_6+a_7+a_8+a_9+a_{11}\end{matrix}\right.$$

 

suy ra $a_{10}=a_{11}$

 

Lập luận tương tự ta suy ra được $a_1=a_2=...=a_9=a_{10}$

 

mà tổng cân nặng của cả đàn gà là $759$ nên $a_1=a_2=...=a_9=a_{10}=a_{11}=\frac{759}{11}=69$




#730168 Đề thi thử đội tuyển Olympic 30-4 THPT Ngô Gia Tự - Khối 11

Đã gửi bởi phuc_90 on 05-09-2021 - 20:24 trong Thi HSG cấp Tỉnh, Thành phố. Olympic 30-4. Đề thi và kiểm tra đội tuyển các cấp.

Bài 1: a=b=0 và c=1 thì VT=1>0=VP



#730524 Tìm a;b để tồn tại k nguyên dương thỏa $a_{0}>a_{1...

Đã gửi bởi phuc_90 on 19-09-2021 - 11:19 trong Dãy số - Giới hạn

Cho dãy số $x_{n}$ xác định như sau: $x_{0}=a;x_{1}=b$ và $x_{n+2}=5x_{n+1}-x_{n}$ biết a;b là các số nguyên dương và a>4b và 5b>a. Tìm tất cả các số nguyên dương a;b như thế để tồn tại một chỉ số k ( k nguyên dương) sao cho $x_{0}>x_{1}>x_{2}>...>x_{k-1}>x_{k}=1$

 

Cho $a_0\,\,,\,\, b_0$ là các số nguyên dương thỏa $$\left\{\begin{matrix}a_0>5\\ a_0>4b_0\\ 5b_0>a_0\\ a_{0}^{2}-5a_0b_0+b_{0}^{2}=1\end{matrix}\right.$$

 

Với mọi số nguyên dương $n$ ta đặt  $$\left\{\begin{matrix}a_n=a_0+nb_0\\ b_n=(5n+1)b_0-na_0\end{matrix}\right.$$

 

Khi đó các giá trị $(a_0,b_0),(a_1,b_1),...,(a_n,b_n),...$ là các giá trị $(a,b)$ phải tìm.

 

Ví dụ:

 

Ta xét các trường hợp riêng

 

-   Với $a_0=24 \,,\, b_0=5$ thì các bộ số $(24,5)\,,\, (29,6)\,,\, (34,7)\,,\,...$ sẽ chính là bộ số $(a,b)$ ta cần tìm, lúc này $k=2$

 

Bây giờ thử bộ đầu tiên $(a,b)=(24,5)$ thì $x_0=24>x_1=5>x_2=1$

 

-   Với $a_0=551\,,\, b_0=115$ thì các bộ số $(551,115)\,,\, (666,139)\,,\, (781,163)\,,...$ chính là bộ số $(a,b)$ ta cần tìm, lúc này $k=4$

 

Bây giờ thử bộ thứ 2 là $(a,b)=(666,139)$ thì $x_0=666>x_1=139>x_2=29>x_3=6>x_4=1$




#730676 Đề chọn đội tuyển VMO Hà Tĩnh 2021-2022

Đã gửi bởi phuc_90 on 24-09-2021 - 10:05 trong Thi HSG cấp Tỉnh, Thành phố. Olympic 30-4. Đề thi và kiểm tra đội tuyển các cấp.

 

Kỳ thi chọn đội tuyển dự thi học sinh giỏi quốc gia THPT năm học 2021-2022

22/09/2021

Bài 1. (5 điểm) 

Cho $a\geq 2$ và $x_1,x_2$ là hai nghiệm của phương trình $x^2-ax+1=0$. Đặt $S_n=x_1^n+x_2^n,n=1,2,...$

a) Chứng minh dãy $\begin{Bmatrix} \frac{S_n}{S_{n+1}} \end{Bmatrix}_{n+1}^{+\infty}$ là dãy giảm.

b) Tìm tất cả các giá trị $a$ sao cho $\frac{S_1}{S_2}+\frac{S_2}{S_3}+...+\frac{S_n}{S_{n+1}}>n-1$. với mọi $n=1,2,...$

 

 

Do $x_1,x_2$ là 2 nghiệm của phương trình $x^2-ax+1=0$  nên $x_1+x_2=a$ và $x_1x_2=1$

 

Khi đó $S_{n+2}=x_1^{n+2}+x_2^{n+2}=(x_1+x_2)(x_1^{n+1}+x_2^{n+1})-x_1x_2(x_1^{n}+x_2^{n})=aS_{n+1}-S_n$

 

Ta có $S_2=x_1^2+x_2^2=a^2-2 \geq a=x_1+x_2=S_1$  giả sử $S_{n+1}\geq S_n\,\,,\,\, \forall n\leq k$

 

Suy ra $S_{k+2}=aS_{k+1}-S_k\geq aS_{k+1}-S_{k+1}=(a-1)S_{k+1}\geq S_{k+1}$

 

Theo nguyên lý qui nạp ta chứng minh được $S_{n+1}\geq S_n\,\,,\,\,\forall n\geq 1$

 

a)   Ta có $S^2_{n+1}-S_nS_{n+2}=(x_1^{n+1}+x_2^{n+1})^2-(x_1^{n}+x_2^{n})(x_1^{n+2}+x_2^{n+2})=2-(x_1^{2}+x_2^{2})=4-a^2\leq 0$

 

nên $\frac{S_{n+1}}{S_{n+2}} \leq \frac{S_n}{S_{n+1}}$  mà điều này đúng với mọi $n\geq 1$

 

Suy ra $\begin{Bmatrix} \frac{S_n}{S_{n+1}} \end{Bmatrix}_{n=1}^{+\infty}$  là dãy giảm

 

b)   Do $S_{n+1}\geq S_n\,\,,\,\,\forall n\geq 1$  nên $0<\frac{S_n}{S_{n+1}}\leq 1\,\,,\,\, \forall n\geq 1$

 

Ta có $S_{n+2}=aS_{n+1}-S_n$      suy ra       $\frac{S_{n+1}}{S_{n+2}}=\frac{1}{a-\frac{S_n}{S_{n+1}}}$

 

Mà dãy   $\begin{Bmatrix} \frac{S_n}{S_{n+1}} \end{Bmatrix}_{n=1}^{+\infty}$        là dãy giảm và bị chặn dưới nên hội tụ, đặt $\lim_{n \to \infty }\frac{S_n}{S_{n+1}}=b$  suy ra $0\leq b\leq 1$

 

Phương trình giới hạn $b=\frac{1}{a-b}$  có nghiệm $b=\frac{a-\sqrt{a^2-4}}{2}$ là thỏa điều kiện

 

Khi đó $\frac{S_1}{S_2}+\frac{S_2}{S_3}+...+\frac{S_n}{S_{n+1}}>nb=\frac{n(a-\sqrt{a^2-4})}{2}$

 

Do đó ta chỉ cần tìm $a$ sao cho $\frac{n(a-\sqrt{a^2-4})}{2}\geq n-1\,\,,\,\, \forall n\geq 1$

 

hay  $(2-a+\sqrt{a^2-4})n<2\,\,,\,\, \forall n\geq 1$

 

điều này chỉ xảy ra khi      $2-a+\sqrt{a^2-4}\leq 0$  hay $a=2$




#731202 LÂM ĐỒNG 2022

Đã gửi bởi phuc_90 on 17-10-2021 - 22:59 trong Thi HSG cấp Tỉnh, Thành phố. Olympic 30-4. Đề thi và kiểm tra đội tuyển các cấp.

 

KỲ THI CHỌN HỌC SINH VÀO ĐỘI TUYỀN BỒI DƯỠNG THI HSG QG NĂM 2022
 
Câu 1. (3.0 điểm) Giải phương trình sau trên tập số thực:
$$2x^3-x^2+\sqrt[3]{2x^3-3x+1}=3x+1+\sqrt[3]{x^2+2}$$
 
Câu 2. (4.0 điểm) Đặt $f(n)=(n^2+n+1)^2+1$. Cho $a_n=\frac{f(1).f(3)...f(2n-1)}{f(2).f(4)...f(2n)}$ với $n$ là số nguyên dương. Chứng minh rằng $\lim n\sqrt{a_n}=\frac{1}{\sqrt{2}}$

 

 

Câu 1:   Đặt $u=\sqrt[3]{2x^3-3x+1}$ và $v=\sqrt[3]{x^2+2}$ thì phương trình trên trở thành $u^3-v^3+u-v=0$

 

hay $(u-v)(u^2+uv+v^2+1)=0$ , vì $u^2+uv+v^2+1=(u+\frac{v}{2})^2+\frac{3v^2}{4}+1>0$

 

nên $u=v$ hay $2x^3-3x+1=u^3=v^3=x^2+2 \Rightarrow 2x^3-x^2-3x-1=0 \Rightarrow (2x+1)(x^2-x-1)=0$

 

Phương trình này cho ta 3 nghiệm $x_1=-\frac{1}{2}\,\,,\,\,x_2=\frac{1+\sqrt{5}}{2}\,\,,\,\,x_3=\frac{1-\sqrt{5}}{2}$

 

Câu 2:   Ta có $f(n)=(n^2+n+1)^2+1=(n^2+1)^2+2n(n^2+1)+n^2+1=(n^2+1)((n+1)^2+1)$

 

Khi đó  $a_n=\frac{f(1).f(3)...f(2n-1)}{f(2).f(4)...f(2n)}$

 

$=\frac{(1^2+1)(2^2+1)(3^2+1)...((2n-1)^2+1)((2n)^2+1)}{(2^2+1)(3^2+1)(4^2+1)(5^2+1)...((2n)^2+1)((2n+1)^2+1)}$

 

$=\frac{2}{(2n+1)^2+1}$

 

Suy ra  $\lim_{n \to \infty } n\sqrt{a_n}=\lim_{n \to \infty }\sqrt{\frac{2n^2}{(2n+1)^2+1}}=\frac{1}{\sqrt{2}}$
 




#730627 $$a_1=\frac{5}{3}\,\,,\,...

Đã gửi bởi phuc_90 on 22-09-2021 - 17:26 trong Dãy số - Giới hạn

Bài toán:   Cho dãy số thực $(a_n)_n$  được xác định như sau

 

$$a_1=\frac{5}{3}\,\,,\,\,a_{n+1}=\frac{1}{4-3a_n}\,\,,\,\,\forall n\geq 1$$

 

Khẳng định hay phủ định $(a_n)_n$ là dãy hội tụ ? Chứng minh nhận định trên.




#730343 Đề thi chọn đội tuyển tỉnh Đồng Tháp 2021

Đã gửi bởi phuc_90 on 12-09-2021 - 21:33 trong Thi HSG cấp Tỉnh, Thành phố. Olympic 30-4. Đề thi và kiểm tra đội tuyển các cấp.

Bài 1:  Cho các số thực $x,y,z$ thỏa mãn $\left\{\begin{matrix}x+y+z=-1\\ x^3+y^3+z^3=11\end{matrix}\right.$

 

       a)   Biểu diễn $zx$ theo y

 

       b)  Chứng minh rằng trong 3 số $x,y,z$ có ít nhất 1 số thuộc đoạn $[-2,-1]$

 

Bài 2:   Cho $a,b,c$ là các số nguyên dương, trong đó $a,b$ nguyên tố cùng nhau và $\frac{b}{c}+\frac{c}{a}$ là số nguyên.

 

Chứng minh rằng $a$ là số chính phương

 

Bài 3:   Cho dãy số $(a_n)_n$ được xác định như sau $\left\{\begin{matrix}a_0=1 \, \, , \, \,a_1=13\\ a_{n+2}=14a_{n+1}-a_n \, \, , \forall n\in \mathbb{N}\end{matrix}\right.$

 

       a)   Chứng minh rằng $2a_n-1$ là số chính phương với mọi số tự nhiên $n$

 

       b)   Chứng minh rằng $a_n$ luôn được viết dưới dạng tổng bình phương của 2 số tự nhiên với mọi số tự nhiên $n$

 

Bài 4:   Tìm tất cả hàm số lẻ $f:\mathbb{R}\rightarrow \mathbb{R}$ thỏa mãn

 

$$f(f(x)+y)=2x+f(x-f(y)) \, \, , \, \, \forall x,y\in \mathbb{R}$$

 

Bài 5:   Cho hai đường tròn $(O_1, R_1)$ và $(O_2,R_2)$ cắt nhau tại $A, B$ sao cho tam giác $AO_1O_2$ vuông tại $A$. Tia $O_1O_2$ cắt đường tròn $(O_2)$ tại $E,F$ và cắt đường tròn $(O_1)$ tại $D$. Điểm $M$ thay đổi trên đường tròn $(O_1)$ và không thuộc đường thẳng $O_1O_2$. Kẻ đường kính $MP$ của $(O_1)$. Tia $O_2M$ cắt đường tròn $(O_1)$ tại điểm thứ 2 là $N$. Tia $O_2P$ cắt đường tròn $(O_1)$ tại điểm thứ 2 là $Q$. Chứng minh rằng :

 

        a)   $MD$ là phân giác của góc $\widehat{EMF}$

 

        b)   $MP, \,NQ, \,AB$ đồng quy hoặc đôi một song song

 

        c)   $NQ$ luôn đi qua 1 điểm cố định

 

Bài 6:   Có 2021 viên bi, đựng trong 100 cái hộp. Mỗi lần, cho phép lấy 2 viên bi, 2 viên bi đó thuộc vào tối đa 2 hộp và bỏ chúng vào 1 hộp khác. Chứng minh rằng sau một số bước có thể bỏ tất cả các viên bi vào cùng 1 hộp.




#735153 Cho $x,y,z$ là các số không âm. Chứng minh rằng: $4(xy+yz+zx)...

Đã gửi bởi phuc_90 on 30-09-2022 - 13:40 trong Tài liệu, chuyên đề, phương pháp về Số học

Gợi ý, áp dụng BĐT Cauchy - Schwarz :  $(ab+cd)^2 \leq (a^2+c^2)(b^2+d^2)$




#730037 Tìm inf, sup của $A=\left \{ \frac{1}{2n}:n= 1,2,......

Đã gửi bởi phuc_90 on 31-08-2021 - 21:25 trong Giải tích

Câu 1, dãy 1/2n đơn điệu giảm bị chặn dưới bởi 0, nên hội tụi và giới hạn của 1/2n bằng 0 là inf của dãy. Hơn nữa vì dãy là hội tụ nên sup của dãy bằng inf của dãy, vậy sup = 0

Sup mà bằng Inf thì tập cần tính chỉ chứa các phần tử bằng nhau và bằng giá trị Sup, Inf, theo đề bài đã cho thì không phải nha bạn.

 

$A=\left \{ \frac{1}{2n} | n\in\mathbb{N^*} \right \}$  có $infA=0$ và $supA=\frac{1}{2}$




#730039 Tính $P(0)$

Đã gửi bởi phuc_90 on 31-08-2021 - 22:28 trong Đa thức

Theo đề bài ta có $\left ( x-1 \right )^2\leq P(x)-2\leq 15\left ( x-1 \right )^2$ với mọi $x$  (*)

 

Cho $x=1$ thì $0 \leq P(1)-2 \leq 0$  suy ra  $P(1)=2$

 

Đặt $G(x)=P(x)-2$ nên $degG=degP=2$ và $G(1)=P(1)-2=0$ hay $1$ là nghiệm của $G(x)$

 

Khi đó $G(x)$ được viết lại thành $G(x)=a(x-1)(x-b)$ với $a,b \in \mathbb{R} $

 

Từ (*) ta có $\left ( x-1 \right )^2\leq a(x-1)(x-b), \forall x$  hay $\left ( x-1 \right )\left ( \left ( a-1 \right )x+1-ab \right )\geq 0 , \forall x$ (**)

 

Từ (**) cho $x=b$ ta có $-(b-1)^2 \geq 0$ suy ra $b=1$, do đó $G(x)=a(x-1)^2$

 

Ta lại có $2016=P(13)-2=G(13)=144a$  suy ra $a=14$

 

Khi đó $P(0)=G(0)+2=14+2=16$

 

 




#731427 Chứng minh ma trận khả nghịch

Đã gửi bởi phuc_90 on 02-11-2021 - 13:10 trong Tài liệu và chuyên đề Đại số tuyến tính và Hình học giải tích

Post hết lời giải lên mới biết được em.



#731191 Với các số thực a, b, c thỏa mãn $1 \leq a, b, c \leq 2$,...

Đã gửi bởi phuc_90 on 17-10-2021 - 15:22 trong Bất đẳng thức và cực trị

Giả sử $a\leq b\leq c$.

Ta có $\frac{(b-a)(b-c)}{ab}\Rightarrow \frac{b}{a}+\frac{c}{b}\leq 1+\frac{c}{a}$.

Tương tự $\frac{(b-a)(b-c)}{bc}\leq 0\Rightarrow \frac{b}{c}+\frac{a}{b}\leq 1+\frac{a}{c}$.

Do đó $\left ( \frac{1}{a}+\frac{1}{b}+\frac{1}{c} \right )(a+b+c)\leq 5+2\frac{a}{c}+2\frac{c}{a}$.

Suy ra $VT\leq \left(\frac{2a}{c}+\frac{2c}{a}+5\right)\left(1+\frac{1}{a}+\frac{1}{c}\right)=7\left(\frac{1}{a}+\frac{1}{c}\right)+\frac{2a}{c^2}+\frac{2c}{a^2}+\frac{2a}{c}+\frac{2c}{a}+5\leq 7\left(\frac{1}{a}+\frac{1}{c}\right)+\frac{2a}{c^2}+\frac{2c}{a^2}+2(a+c)+5=A+5$.

Ta lại có $\frac{(a-1)(a-2)}{a}\leq 0\Rightarrow a\leq 3-\frac{2}{a}$. Tương tự $c\leq 3-\frac{2}{c}$ nên $A\leq 12+3\left(\frac{a}{c}+\frac{c}{a}\right)+\frac{2c}{a^2}+\frac{2a}{c^2}$.

Ta chứng minh: $3\left ( \frac{a}{c}+\frac{c}{a} \right )+\frac{2c}{a^2}+\frac{2a}{c^2}\leq 3+\frac{3}{c}+\frac{2}{c^2}+2c$. (*)

$(*)\Leftrightarrow 2c\left ( 1-\frac{1}{a^2} \right )+3\left ( 1-\frac{1}{a} \right )\geq \frac{2}{c^2}(a-1)$

$\Leftrightarrow \left ( a-1 \right )\left ( \frac{2c(a+1)}{a^2}+\frac{3}{a}-\frac{2}{c^2} \right )\geq 0$. (luôn đúng do $a-1\geq 0$ và $\frac{2c(a+1)}{a^2}+\frac{3}{a}-\frac{2}{c^2}=2c\left ( \frac{1}{a}+\frac{1}{a^2} \right )+\frac{3}{a}-\frac{2}{c^2}\geq \frac{3c}{2}+\frac{3}{2}-\frac{2}{c^2}>0$).

Suy ra ta chỉ cần chứng minh $3+\frac{3}{c}+\frac{2}{c^2}+2c\leq 10\Leftrightarrow \frac{(c-1)(2c^2-5c-2)}{c^2}\leq 0$. (luôn đúng)

Do đó $A\leq 12+10=22\Rightarrow VT\leq 27$.

Đẳng thức xảy ra khi và chỉ khi $a=b=c=1$.

 

Chỗ chữ màu xanh hình như có vấn đề ?

 

Ta luôn có $\frac{a}{c}+\frac{c}{a}\geq \frac{1}{a}+\frac{1}{c}$




#730418 $x_0=\frac{1}{2} , x_{n+1}=x_n+\...

Đã gửi bởi phuc_90 on 15-09-2021 - 15:52 trong Dãy số - Giới hạn

let f = (x, b) => x + x*x / b;
function findLB(a, b, c) {
    let x = a;
    let n = 0;
    while (x <= c) {
        n++;
        x = f(x, b);
    }
    return n;
}

Vọc code một tí thì có thể thấy $n=2019$ và nói chung là với mọi $n$ thì $k=n+2$ sẽ là GTNN để $x_k > 1$.

 

Vậy khi viết lời giải bài toán này ra giấy thì ghi như thế nào ?  :o




#729886 $\sum a\left ( a-b \right )\left ( a-c \right )...

Đã gửi bởi phuc_90 on 23-08-2021 - 15:45 trong Bất đẳng thức - Cực trị

Chuẩn hóa $a+b+c=3$. Đặt $bc+ca+ab=3-3t^{2}$. Khi đó $0 \leq r=$$abc\leq (1+2t)(1-t)^{2}$.

Bất đẳng thức tương đương $$(144t^{2}-27)r+27(2t-1)^{2}(2t+1)^{2}\geq 0.$$

Nếu $144t^{2}\geq 27$, bất đẳng thức là hiển nhiên. Xét $144t^{2}<27$.

Khi đó $$VT\geq (144t^{2}-27)(1+2t)(1-t)^{2}+27(2t-1)^{2}(2t+1)^{2}=9t^{2}(2t+1)(4t-1)^{2}\geq 0.$$

Vậy bất đẳng thức đề cho là đúng.

Đẳng thức xảy ra khi $a=b=c$ hoặc $a=0,b=c$ và các hoán vị, hoặc $a=2b=2c$ và các hoán vị. $\square$

Cái chỗ màu xanh làm rõ hơn được không bạn ?




#730353 $x_0=\frac{1}{2} , x_{n+1}=x_n+\...

Đã gửi bởi phuc_90 on 13-09-2021 - 14:59 trong Dãy số - Giới hạn

Bài toán:  Cho dãy số thực $(x_n)_n$ được định nghĩa như sau

 

$$x_0=\frac{1}{2} \, \, , \, \, x_{n+1}=x_n+\frac{x^{2}_{n}}{2017} \,\, , \,\, \forall n\in \mathbb{N}$$

 

Tìm số tự nhiên $k$ nhỏ nhất sao cho $x_k>1$

 

Note:  Đây là bài toán cũ không có lời giải của diễn đàn, thấy thú vị nên post lại cho các bạn giải thử. Mình thì tìm được $k=4034$, bạn nào tìm được giá trị nhỏ hơn thì cho lời giải nhé.




#729880 $\sum a\left ( a-b \right )\left ( a-c \right )...

Đã gửi bởi phuc_90 on 23-08-2021 - 09:56 trong Bất đẳng thức - Cực trị

Cho $a,b,c$ là các số thực không âm. Chứng minh rằng:

 

$a\left ( a-b \right )\left ( a-c \right )\left ( a-2b \right )\left ( a-2c \right )+b\left ( b-c \right )\left ( b-a \right )\left ( b-2c \right )\left ( b-2a \right )+c\left ( c-a \right )\left ( c-b \right )\left ( c-2a \right )\left ( c-2b \right ) \geq  0$